Proof of $A\subseteq B\implies\sup A\le\sup B$

  • MHB
  • Thread starter alexmahone
  • Start date
  • Tags
    Proof
In summary, the proof shows that if $A$ is a subset of $B$, and $B$ has an upper bound $\sup B$, then $\sup B$ is also an upper bound for $A$. Since $\sup A$ is the least upper bound of $A$, it follows that $\sup A\le\sup B$. This proof assumes that $A$ and $B$ are non-empty subsets of $\mathbb{R}$ bounded above, and is supported by the Axiom of Completeness.
  • #1
alexmahone
304
0
Prove: $A\subseteq B\implies\sup A\le\sup B$

---------- Post added at 03:21 PM ---------- Previous post was at 03:03 PM ----------

By the definition of subset,

$x\in A\implies x\in B$

$\sup B$ is an upper bound of $B$.

$x\in B\implies x\le\sup B$

So, $x\in A\implies x\le\sup B$

ie $\sup B$ is an upper bound of $A$.

But $\sup A$ is the least upper bound of $A$.

So, $\sup A\le\sup B$

-----------------------------------------------------

Is the above proof ok?
 
Physics news on Phys.org
  • #2
Alexmahone said:
Prove: $A\subseteq B\implies\sup A\le\sup B$

---------- Post added at 03:21 PM ---------- Previous post was at 03:03 PM ----------

By the definition of subset,

$x\in A\implies x\in B$

$\sup B$ is an upper bound of $B$.

$x\in B\implies x\le\sup B$

So, $x\in A\implies x\le\sup B$

ie $\sup B$ is an upper bound of $A$.

But $\sup A$ is the least upper bound of $A$.

So, $\sup A\le\sup B$

-----------------------------------------------------

Is the above proof ok?
Yes, but above your prove, say a word about the Axiom of completeness.
 
  • #3
this looks fine to me. But the problem should state that A and B are non empty subsets of \( \mathbb{R}\) that are bounded above. Only in that case , it makes sense to talk about supremum's...

Edit: it seems somebody already said what I wanted to...hmm
 

Related to Proof of $A\subseteq B\implies\sup A\le\sup B$

1. What does "Proof of $A\subseteq B\implies\sup A\le\sup B$" mean?

"Proof of $A\subseteq B\implies\sup A\le\sup B$" is a mathematical statement that states if all elements in set A are also elements in set B, then the supremum (or least upper bound) of set A is less than or equal to the supremum of set B.

2. How do you prove that $A\subseteq B\implies\sup A\le\sup B$?

To prove that $A\subseteq B\implies\sup A\le\sup B$, you can use the definition of supremum, which states that the supremum of a set is the least upper bound. You can also use the fact that if all elements in set A are also elements in set B, then the supremum of set B must be greater than or equal to the supremum of set A.

3. What is the significance of proving $A\subseteq B\implies\sup A\le\sup B$?

Proving $A\subseteq B\implies\sup A\le\sup B$ is significant because it helps to establish the order and relationship between sets. It also allows for the comparison of the supremum of two sets, which can be useful in various mathematical and scientific applications.

4. Can you give an example of a proof for $A\subseteq B\implies\sup A\le\sup B$?

One example of a proof for $A\subseteq B\implies\sup A\le\sup B$ is as follows: Let x be an element in set A. Since A is a subset of B, x is also an element in set B. Therefore, the supremum of set B must be greater than or equal to x. Since x was chosen arbitrarily, this holds true for all elements in set A. Thus, the supremum of set B must be greater than or equal to the supremum of set A, proving $A\subseteq B\implies\sup A\le\sup B$.

5. Are there any other similar statements related to $A\subseteq B\implies\sup A\le\sup B$?

Yes, there are other similar statements related to $A\subseteq B\implies\sup A\le\sup B$. One example is $A\subseteq B\implies\inf B\le\inf A$, which states that if all elements in set A are also elements in set B, then the infimum (or greatest lower bound) of set B is greater than or equal to the infimum of set A. This is the dual statement of $A\subseteq B\implies\sup A\le\sup B$, where the roles of supremum and infimum are reversed.

Similar threads

Replies
1
Views
846
Replies
2
Views
3K
  • Topology and Analysis
Replies
1
Views
987
Replies
2
Views
1K
Replies
2
Views
3K
Replies
2
Views
2K
Replies
5
Views
1K
Replies
2
Views
1K
  • Calculus and Beyond Homework Help
Replies
1
Views
1K
Replies
4
Views
1K
Back
Top